matheraum.de
Raum für Mathematik
Offene Informations- und Nachhilfegemeinschaft

Für Schüler, Studenten, Lehrer, Mathematik-Interessierte.
Hallo Gast!einloggen | registrieren ]
Startseite · Forum · Wissen · Kurse · Mitglieder · Team · Impressum
Forenbaum
^ Forenbaum
Status Hochschulmathe
  Status Uni-Analysis
    Status Reelle Analysis
    Status UKomplx
    Status Uni-Kompl. Analysis
    Status Differentialgl.
    Status Maß/Integrat-Theorie
    Status Funktionalanalysis
    Status Transformationen
    Status UAnaSon
  Status Uni-Lin. Algebra
    Status Abbildungen
    Status ULinAGS
    Status Matrizen
    Status Determinanten
    Status Eigenwerte
    Status Skalarprodukte
    Status Moduln/Vektorraum
    Status Sonstiges
  Status Algebra+Zahlentheo.
    Status Algebra
    Status Zahlentheorie
  Status Diskrete Mathematik
    Status Diskrete Optimierung
    Status Graphentheorie
    Status Operations Research
    Status Relationen
  Status Fachdidaktik
  Status Finanz+Versicherung
    Status Uni-Finanzmathematik
    Status Uni-Versicherungsmat
  Status Logik+Mengenlehre
    Status Logik
    Status Mengenlehre
  Status Numerik
    Status Lin. Gleich.-systeme
    Status Nichtlineare Gleich.
    Status Interpol.+Approx.
    Status Integr.+Differenz.
    Status Eigenwertprobleme
    Status DGL
  Status Uni-Stochastik
    Status Kombinatorik
    Status math. Statistik
    Status Statistik (Anwend.)
    Status stoch. Analysis
    Status stoch. Prozesse
    Status Wahrscheinlichkeitstheorie
  Status Topologie+Geometrie
  Status Uni-Sonstiges

Gezeigt werden alle Foren bis zur Tiefe 2

Navigation
 Startseite...
 Neuerdings beta neu
 Forum...
 vorwissen...
 vorkurse...
 Werkzeuge...
 Nachhilfevermittlung beta...
 Online-Spiele beta
 Suchen
 Verein...
 Impressum
Das Projekt
Server und Internetanbindung werden durch Spenden finanziert.
Organisiert wird das Projekt von unserem Koordinatorenteam.
Hunderte Mitglieder helfen ehrenamtlich in unseren moderierten Foren.
Anbieter der Seite ist der gemeinnützige Verein "Vorhilfe.de e.V.".
Partnerseiten
Weitere Fächer:

Open Source FunktionenplotterFunkyPlot: Kostenloser und quelloffener Funktionenplotter für Linux und andere Betriebssysteme
StartseiteMatheForenFourier-TransformationKomplexe Fourier-Reihe
Foren für weitere Studienfächer findest Du auf www.vorhilfe.de z.B. Astronomie • Medizin • Elektrotechnik • Maschinenbau • Bauingenieurwesen • Jura • Psychologie • Geowissenschaften
Forum "Fourier-Transformation" - Komplexe Fourier-Reihe
Komplexe Fourier-Reihe < Fourier-Transformati < Transformationen < Analysis < Hochschule < Mathe < Vorhilfe
Ansicht: [ geschachtelt ] | ^ Forum "Fourier-Transformation"  | ^^ Alle Foren  | ^ Forenbaum  | Materialien

Komplexe Fourier-Reihe: Frage (beantwortet)
Status: (Frage) beantwortet Status 
Datum: 21:30 Sa 27.10.2018
Autor: hase-hh

Aufgabe
Es soll im folgenden eine Fourier-Reihe gebildet werden.

Ich bin allerdings nicht sicher, ob ich hier im richtigen Forum/Unterforum gelandet bin???



Entwickeln Sie die komplexe Fourierreihe der Funktion

f(x) = x  [mm] (-\pi \le [/mm] x < [mm] \pi) [/mm]


Moin Moin,

ich habe hier (weiter unten) eine Frage zum Einheitskreis...

Also zunächst ist eine komplexe Fourierreihe definiert als

F f(x)  = [mm] \summe_{n=-\infty}^{+\infty} c_n*e^{inx} [/mm]

[mm] c_n [/mm] = [mm] \bruch{1}{2\pi}*\integral_{0}^{2\pi}{f(x)*e^{-inx} dx} [/mm]

Hier hätte ich schon mal ne Frage: Was bedeutet F f(x) ???


1. [mm] c_n [/mm] bestimmen

[mm] c_n [/mm] = [mm] \bruch{1}{2\pi}*\integral_{0}^{2\pi}{f(x)*e^{-inx} dx} [/mm]

Wegen der Unstetigkeitsstelle bei [mm] \pi [/mm] muss das Integral aufgeteilt werden...

[mm] c_n [/mm] = [mm] \bruch{1}{2\pi}*\integral_{0}^{\pi}{x*e^{-inx} dx} [/mm] + [mm] \bruch{1}{2\pi}*\integral_{\pi}^{2\pi}{(x-2\pi) *e^{-inx} dx} [/mm]


f(x) = x  ist die Gerade durch (0/0), die bis  [mm] (\pi/\pi) [/mm] geht
g(x) = x - [mm] 2\pi [/mm] ist die Gerade, die bei [mm] (\pi/-\pi) [/mm] anfängt über [mm] (2\pi/0) [/mm] bis [mm] (3\pi/\pi) [/mm] geht... usw.

Aber wir betrachten ja nur das Intervall [mm] [0;2\pi]. [/mm]

Ich hoffe, das ist soweit richtig?!

Wir fassen zusammen
[mm] c_n [/mm] = [mm] \bruch{1}{2\pi}*\integral_{0}^{2\pi}{xe^{-inx} dx} +\bruch{1}{2\pi}*\integral_{\pi}^{2\pi}{-2\pi*e^{-inx} dx} [/mm]

[mm] c_n [/mm] = [mm] \bruch{1}{2\pi}*\integral_{0}^{2\pi}{x*e^{-inx} dx} [/mm] - [mm] \integral_{\pi}^{2\pi}{e^{-inx} dx} [/mm]


Nun wird der erste Summand mit partieller Integration gebildet...

u = x    v' = [mm] e^{-inx} [/mm]

u' = 1   v = [mm] -\bruch{1}{in}*e^{-inx} [/mm]

[mm] c_n [/mm] = [mm] \bruch{1}{2\pi}*[x*(-\bruch{1}{in}*e^{-inx})] [/mm] - [mm] \bruch{1}{2\pi}*\integral_{0}^{2\pi}{1*(-\bruch{1}{in}*e^{-inx}) dx} [/mm] - [mm] \integral_{\pi}^{2\pi}{e^{-i*n*x} dx} [/mm]

[mm] c_n [/mm] = [mm] \bruch{1}{2\pi}*[x*(-\bruch{1}{in}*e^{-inx})] \vmat{ 0 \\ 2\pi } [/mm] - [mm] \bruch{1}{2\pi}*[(+\bruch{1}{(in)^2}*e^{-inx}] \vmat{ 0 \\ 2\pi } [/mm] - [mm] [-\bruch{1}{in}*e^{-i*n*x}] \vmat{ \pi \\ 2\pi } [/mm]

[mm] c_n [/mm] = [mm] (-\bruch{1}{in}*e^{-in*2\pi} [/mm] - 0) [mm] -(\bruch{1}{2\pi}*\bruch{1}{(in)^2}*e^{-in*2\pi} [/mm] - [mm] \bruch{1}{2\pi}*\bruch{1}{(in)^2}*e^{-in*0}) [/mm] - [mm] (-\bruch{1}{in}*e^{-in*2\pi} [/mm] - [mm] (-\bruch{1}{in}*e^{-in*\pi})) [/mm]


Jetzt wird gesagt, dass [mm] e^{-i*n*2\pi} [/mm]  = 1

und [mm] e^{-i*n*\pi} [/mm] = -1 sei. Und dies wird mit dem Einheitskreis begründet.

Aber wie hängt die e-Funktion bzw. diese e-Funktion mit dem Einheitskreis zusammen???

sin(x)  ok; cos(x)  ok; tan(x)  ok... aber [mm] e^x [/mm]  ???

Kann mir das mal jemand erklären???


Danke & Gruß!



        
Bezug
Komplexe Fourier-Reihe: Antwort
Status: (Antwort) fertig Status 
Datum: 14:12 So 28.10.2018
Autor: Gonozal_IX

Hiho,

> Also zunächst ist eine komplexe Fourierreihe definiert als
>
> F f(x)  = [mm]\summe_{n=-\infty}^{+\infty} c_n*e^{inx}[/mm]
>  
> [mm]c_n[/mm] = [mm]\bruch{1}{2\pi}*\integral_{0}^{2\pi}{f(x)*e^{-inx} dx}[/mm]

Das ist interessant, woher hast du das?
Ich kenn das bisher nur als

[mm]c_n[/mm] = [mm]\bruch{1}{2\pi}*\integral_{-\pi}^{\pi}{f(x)*e^{-inx} dx}[/mm]
was auch dein späteres "Problem" mit dem Shift lösen würde… deine Funktion ist nämlich gerade im Bereich [mm] $-\pi \le [/mm] x [mm] \le \pi$ [/mm] definiert und muss dann gar nicht verschoben werden… ich schreibe "Problem" weil es eigentlich gar keins ist… dann kommt nämlich faktisch dasselbe raus.
  

> Hier hätte ich schon mal ne Frage: Was bedeutet F f(x)

F ist der Fourieroperator, die Klammern werden gern mal weggelassen, d.h. du erhälst die Fouriertransformierte von f durch Anwendung von F auf f, also faktisch $F(f)$

> Jetzt wird gesagt, dass [mm]e^{-i*n*2\pi}[/mm]  = 1
>
> und [mm]e^{-i*n*\pi}[/mm] = -1 sei. Und dies wird mit dem
> Einheitskreis begründet.
>  
> Aber wie hängt die e-Funktion bzw. diese e-Funktion mit
> dem Einheitskreis zusammen???
>
> sin(x)  ok; cos(x)  ok; tan(x)  ok... aber [mm]e^x[/mm]  ???
>  
> Kann mir das mal jemand erklären???

Nicht [mm] $e^x$ [/mm] sondern [mm] $e^{ix}$. [/mm]
Komplexe Zahlen können einerseits geschrieben werden als $z = x + iy$ aber auch als [mm] $z=re^{i\varphi}$ [/mm] wobei dann [mm] $r=\sqrt{x^2 + y^2}=|z|$ [/mm] der Betrag der Komplexen Zahl ist.

Man erkennt sofort, dass dann für alle Zahlen der Form [mm] $z=1*e^{i\varphi}$ [/mm] der Betrag der Zahl gleich 1 ist.

D.h. die Abbildung [mm] $\varphi \mapsto e^{i\varphi}$ [/mm] bildet also auf den Einheitskreis in den Komplexen Zahlen ab, insbesondere gilt also für jedes [mm] $\varphi \in \IR$ [/mm] dann $|z| = [mm] \left|e^{i\varphi}\right| [/mm] = 1$ und [mm] $\varphi$ [/mm] beschreibt den Winkel zwischen der reellen Achse und der komplexen Zahl.

Weiterhin gilt (siehe auch []hier) dann die eulersche Formel: [mm] ${\displaystyle \mathrm {e} ^{\mathrm {i} \,\varphi}=\cos \left(\varphi\right)+\mathrm {i} \,\sin \left(\varphi\right)}$. [/mm] In dem Artikel siehst du auch eine Grafik, die das oben mit dem Winkel nochmal schön erklärt. Durch obige Identität, aber auch am Bild erkennt man dann eben sofort, dass gilt:
$1 = 1 + 0i = [mm] e^{i\cdot 0}$ [/mm] sowie $-1 = -1 + 0i = [mm] e^{-i\pi}$ [/mm]

Gruß,
Gono

Bezug
                
Bezug
Komplexe Fourier-Reihe: Mitteilung
Status: (Mitteilung) Reaktion unnötig Status 
Datum: 15:56 So 28.10.2018
Autor: hase-hh

Moin Moin,

zunächst zu deiner Frage...

>  
> Das ist interessant, woher hast du das?
>  Ich kenn das bisher nur als
>
> [mm]c_n[/mm] = [mm]\bruch{1}{2\pi}*\integral_{-\pi}^{\pi}{f(x)*e^{-inx} dx}[/mm]
>  
> was auch dein späteres "Problem" mit dem Shift lösen
> würde… deine Funktion ist nämlich gerade im Bereich
> [mm]-\pi \le x \le \pi[/mm] definiert und muss dann gar nicht
> verschoben werden… ich schreibe "Problem" weil es
> eigentlich gar keins ist… dann kommt nämlich faktisch
> dasselbe raus.

Äh, das habe ich aus einem youtube-Video:

https://www.youtube.com/watch?v=9JlfTbckvCg




Bezug
                        
Bezug
Komplexe Fourier-Reihe: Mitteilung
Status: (Mitteilung) Reaktion unnötig Status 
Datum: 16:47 So 28.10.2018
Autor: Gonozal_IX

Hiho,

> Äh, das habe ich aus einem youtube-Video:
>
> https://www.youtube.com/watch?v=9JlfTbckvCg

gut, dass du das gezeigt hast… du hast nämlich ein Verständnisproblem!
Dort wird nämlich nicht die Fouriertransformierte von deinem gegebenem f gebildet.

Gegeben ist die [mm] $2\pi$-periodische [/mm] Funktion $f$, welche eingeschränkt auf das Intervall [mm] $[-\pi,\pi)$ [/mm] die Gestalt $f(x)=x$ hat, also in Formeln:
[mm] $f(x)\big|_{[-\pi,\pi)} [/mm] = x$ hat.

Und nun sollst du die Fouriertransformierte auf dem Intervall [mm] $[0,2\pi)$ [/mm] bilden.

Dazu überlegst du dir, das f eingeschränkt auf [mm] $[0,2\pi]$ [/mm] die Gestalt hat

[mm] $f(x)\big|_{[0,2\pi)} [/mm] = [mm] \begin{cases} 0, & 0 \le x < \pi \\ x-2\pi, & \pi \le x < 2\pi \end{cases}$ [/mm]

Das erklärt auch, warum nachher die Integrale so aussehen, wie sie aussehen.
Nix mit Transformationen o.Ä.

Bei deiner Aufgabenstellung mit der Funktion

> f(x) = x  $ [mm] (-\pi \le [/mm] $ x < $ [mm] \pi) [/mm] $

könnte man denken, es wäre $f(x) = [mm] x1_{[-\pi,\pi)}$ [/mm] und damit wäre f nämlich Null für $x [mm] \ge \pi$. [/mm]

Wenn es dir natürlich klar ist, dass du ausschließlich [mm] $2\pi$-periodische [/mm] Funktionen betrachtest, solltest du das trotzdem hinschreiben.
Das macht man korrekterweise zu Beginn auch so, kann man später aber verallgemeinern ;-)

Gruß,
Gono



Bezug
                
Bezug
Komplexe Fourier-Reihe: Mitteilung
Status: (Mitteilung) Reaktion unnötig Status 
Datum: 17:34 So 28.10.2018
Autor: hase-hh

...
Bezug
                
Bezug
Komplexe Fourier-Reihe: Frage (beantwortet)
Status: (Frage) beantwortet Status 
Datum: 17:36 So 28.10.2018
Autor: hase-hh

Vielen Dank für die ausführliche Antwort!


1. Ich berechne als erstes [mm] c_0 [/mm]

[mm] c_0 [/mm] = [mm] \bruch{1}{2\pi}*\integral_{-\pi}^{\pi}{x*e^{-in*0} dx} [/mm]  

[mm] c_0 [/mm] = [mm] \bruch{1}{2\pi}*\integral_{-\pi}^{\pi}{x*1 dx} [/mm]  

[mm] c_0 [/mm] = [mm] \bruch{1}{2\pi}*[\bruch{1}{2}*x^2] [/mm]   von [mm] -\pi [/mm] bis [mm] \pi [/mm]


[mm] c_0 [/mm] = 0


2. Ich berechne die [mm] c_n [/mm]


[mm] c_n [/mm] = [mm] \bruch{1}{2\pi}*\integral_{-\pi}^{\pi}{x*e^{-inx} dx} [/mm]  

Ich wähle für die partielle Integration

u = x       v' = [mm] e^{-inx} [/mm]

u' = 1      v = [mm] -\bruch{1}{in}*e^{-inx} [/mm]

[mm] c_n [/mm] = [mm] \bruch{1}{2\pi}*( [-\bruch{1}{in}*x*e^{-inx}] [/mm] - [mm] \integral_{-\pi}^{\pi}{-\bruch{1}{in}*e^{-inx} dx}) [/mm]

[mm] c_n [/mm] = [mm] \bruch{1}{2\pi}*( [-\bruch{1}{in}*x*e^{-inx}] [/mm] - [mm] [(-\bruch{1}{in})^2*e^{-inx}]) [/mm]

[mm] c_n [/mm] = [mm] \bruch{1}{2\pi}*(-\bruch{1}{in}*\pi*e^{(-i*\pi)^n} -(-\bruch{1}{in}*(-\pi)*e^{-i*(-\pi)^n})) [/mm] - [mm] [\bruch{1}{i^2n^2}*e^{-inx}] [/mm]

[mm] c_n [/mm] = [mm] \bruch{1}{2\pi}*(-\bruch{\pi}{in}*(-1)^n -(-\bruch{-\pi}{in}*(-1)^n)) [/mm] - [mm] [\bruch{1}{-1*n^2}*e^{-inx}] [/mm]

[mm] c_n [/mm] = [mm] \bruch{1}{2\pi}*(-\bruch{\pi}{in}*(-1)^n -(\bruch{\pi}{in}*(-1)^n)) [/mm] + [mm] [\bruch{1}{n^2}*e^{-inx}] [/mm]


[mm] c_n [/mm] =  [mm] \bruch{1}{2\pi}*(\bruch{-2*\pi}{in}*(-1)^n) [/mm] + [mm] (\bruch{1}{n^2}*e^{(-i*\pi)^n} [/mm] - [mm] \bruch{1}{n^2}*e^{-i*(-\pi))^n}) [/mm]

[mm] c_n [/mm] =  (- [mm] \bruch{2}{in}*(-1)^n) [/mm] + [mm] (\bruch{1}{n^2}*(-1)^n [/mm] - [mm] \bruch{1}{n^2}*(-1)^n) [/mm]

[mm] c_n [/mm] =  (- [mm] \bruch{2}{in}*(-1)^n) [/mm]  + 0


Ist das richtig?


Bezug
                        
Bezug
Komplexe Fourier-Reihe: Antwort
Status: (Antwort) fertig Status 
Datum: 18:31 Di 30.10.2018
Autor: Gonozal_IX

Hiho,

entschuldige die späte Antwort, aber ich war etwas dahingerafft von kleinen, widerspenstigen Lebewesen…

> 1. Ich berechne als erstes [mm]c_0[/mm]
> […]
> [mm]c_0[/mm] = 0

[ok]

>
> 2. Ich berechne die [mm]c_n[/mm]
>
>
> [mm]c_n[/mm] = [mm]\bruch{1}{2\pi}*\integral_{-\pi}^{\pi}{x*e^{-inx} dx}[/mm]
>  
>
> Ich wähle für die partielle Integration
>
> u = x       v' = [mm]e^{-inx}[/mm]
>  
> u' = 1      v = [mm]-\bruch{1}{in}*e^{-inx}[/mm]

[ok]
  

> [mm]c_n[/mm] = [mm]\bruch{1}{2\pi}*( [-\bruch{1}{in}*x*e^{-inx}][/mm] -
> [mm]\integral_{-\pi}^{\pi}{-\bruch{1}{in}*e^{-inx} dx})[/mm]

Hier hast du im ersten Ausdruck die Grenzen vergessen…

> [mm]c_n[/mm] = [mm]\bruch{1}{2\pi}*( [-\bruch{1}{in}*x*e^{-inx}][/mm] -
> [mm][(-\bruch{1}{in})^2*e^{-inx}])[/mm]

Hier schreibst du auch wieder keine Grenzen hin.

> [mm]c_n[/mm] = [mm]\bruch{1}{2\pi}*(-\bruch{1}{in}*\pi*e^{(-i*\pi)^n} -(-\bruch{1}{in}*(-\pi)*e^{-i*(-\pi)^n}))[/mm]
> - [mm][\bruch{1}{i^2n^2}*e^{-inx}][/mm]Eingabefehler: "{" und "}" müssen immer paarweise auftreten, es wurde aber ein Teil ohne Entsprechung gefunden (siehe rote Markierung)


Und hier wird aus dem $i\pi n$ im Exponent plötzlich ein $i\pi^n$… wieso?

Dein Ansatz ist aber ok… allerdings ist mir aufgefallen, dass meine Aussage zu Beginn falsch war, dass der Integrationsbereich keine Rolle spielt… zumindest für die Lösung.
Du müsstest jetzt bei $e^{-in\pi}$ nämlich zwischen geraden und ungeraden n unterscheiden.
Diese Unterscheidung fällt praktischerweise beim Integrationsbereich von 0 bis $2\pi$ weg, weil der Faktor 2 da immer drin vorkommt.

D.h. bezugnehmend auf meinen Kommentar solltest du korrekterweise $ c_n = \bruch{1}{2\pi}\cdot{}\integral_{0}^{2\pi}{x\cdot{}e^{-inx dx} $   berechnen… das war ja auch die Aufgabenstellung.

Gruß,
Gono

Bezug
                                
Bezug
Komplexe Fourier-Reihe: Frage (beantwortet)
Status: (Frage) beantwortet Status 
Datum: 12:59 Mi 31.10.2018
Autor: hase-hh

Moin Moin Gono,

> Hiho,
>  
> entschuldige die späte Antwort, aber ich war etwas
> dahingerafft von kleinen, widerspenstigen Lebewesen…

Ich hoffe, du hast mit deinen Mäusen mittlerweile Einvernehmen erzielt! :-)


> > [mm]c_n[/mm] = [mm]\bruch{1}{2\pi}*( [-\bruch{1}{in}*x*e^{-inx}][/mm] -
> > [mm]\integral_{-\pi}^{\pi}{-\bruch{1}{in}*e^{-inx} dx})[/mm]
> Hier hast du im ersten Ausdruck die Grenzen vergessen…

Naja, ich weiß nicht, wie ich die Grenzen da angeben kann (Formatierungsproblem)   ... [mm] [-\pi;\pi] [/mm]  ist jeweils gemeint...
  

> > [mm]c_n[/mm] = [mm]\bruch{1}{2\pi}*( [-\bruch{1}{in}*x*e^{-inx}][/mm] -
> > [mm][(-\bruch{1}{in})^2*e^{-inx}])[/mm]
> Hier schreibst du auch wieder keine Grenzen hin.

Naja, ich weiß nicht, wie ich die Grenzen da angeben kann (Formatierungsproblem)   ... [mm] [-\pi;\pi] [/mm]  ist jeweils gemeint...


> > [mm]c_n[/mm] = [mm]\bruch{1}{2\pi}*(-\bruch{1}{in}*\pi*e^{(-i*\pi)^n} -(-\bruch{1}{in}*(-\pi)*e^{-i*(-\pi)^n}))[/mm]
> > - [mm][\bruch{1}{i^2n^2}*e^{-inx}][/mm]
>  Und hier wird aus dem [mm]i\pi n[/mm] im Exponent plötzlich ein
> [mm]i\pi^n[/mm]… wieso?

Ich habe [mm] e^{-i*\pi*n} [/mm] umgeformt zu [mm] e^{(-i*\pi)^n} [/mm] ...weil [mm] e^{-i*\pi} [/mm] = -1  bzw. [mm] e^{i*\pi} [/mm] = -1  ist, oder nicht?

>  
> Dein Ansatz ist aber ok… allerdings ist mir aufgefallen,
> dass meine Aussage zu Beginn falsch war, dass der
> Integrationsbereich keine Rolle spielt… zumindest für
> die Lösung.
> Du müsstest jetzt bei [mm]e^{-in\pi}[/mm] nämlich zwischen geraden
> und ungeraden n unterscheiden.

Also hier müsste ich nochmal weitermachen... richtig?

>  Diese Unterscheidung fällt praktischerweise beim
> Integrationsbereich von 0 bis [mm]2\pi[/mm] weg, weil der Faktor 2
> da immer drin vorkommt.
>
> D.h. bezugnehmend auf meinen Kommentar solltest du
> korrekterweise [mm]c_n = \bruch{1}{2\pi}\cdot{}\integral_{0}^{2\pi}{x\cdot{}e^{-inx dx}[/mm]
>   berechnen… das war ja auch die Aufgabenstellung.
>  
> Gruß,
>  Gono  

Naja, die Aufgabenstellung war schon das Intervall [mm] [-\pi;\pi] [/mm] .


Bezug
                                        
Bezug
Komplexe Fourier-Reihe: Antwort
Status: (Antwort) fertig Status 
Datum: 13:25 Mi 31.10.2018
Autor: fred97


> Moin Moin Gono,
>  
> > Hiho,
>  >  
> > entschuldige die späte Antwort, aber ich war etwas
> > dahingerafft von kleinen, widerspenstigen Lebewesen…
>  
> Ich hoffe, du hast mit deinen Mäusen mittlerweile
> Einvernehmen erzielt! :-)
>  
>
> > > [mm]c_n[/mm] = [mm]\bruch{1}{2\pi}*( [-\bruch{1}{in}*x*e^{-inx}][/mm] -
> > > [mm]\integral_{-\pi}^{\pi}{-\bruch{1}{in}*e^{-inx} dx})[/mm]
> > Hier hast du im ersten Ausdruck die Grenzen vergessen…
>  
> Naja, ich weiß nicht, wie ich die Grenzen da angeben kann
> (Formatierungsproblem)   ... [mm][-\pi;\pi][/mm]  ist jeweils


[mm] $F(b)-F(a)=[F(x)]_a^b$ [/mm]

meinst Du das ?

> gemeint...
>    
> > > [mm]c_n[/mm] = [mm]\bruch{1}{2\pi}*( [-\bruch{1}{in}*x*e^{-inx}][/mm] -
> > > [mm][(-\bruch{1}{in})^2*e^{-inx}])[/mm]
> > Hier schreibst du auch wieder keine Grenzen hin.
>  
> Naja, ich weiß nicht, wie ich die Grenzen da angeben kann
> (Formatierungsproblem)   ... [mm][-\pi;\pi][/mm]  ist jeweils
> gemeint...

s.o.


>  
>
> > > [mm]c_n[/mm] = [mm]\bruch{1}{2\pi}*(-\bruch{1}{in}*\pi*e^{(-i*\pi)^n} -(-\bruch{1}{in}*(-\pi)*e^{-i*(-\pi)^n}))[/mm]
> > > - [mm][\bruch{1}{i^2n^2}*e^{-inx}][/mm]
>  >  Und hier wird aus dem [mm]i\pi n[/mm] im Exponent plötzlich ein
> > [mm]i\pi^n[/mm]… wieso?
>  
> Ich habe [mm]e^{-i*\pi*n}[/mm] umgeformt zu [mm]e^{(-i*\pi)^n}[/mm] ...weil
> [mm]e^{-i*\pi}[/mm] = -1  bzw. [mm]e^{i*\pi}[/mm] = -1  ist, oder nicht?

Upps ! Es ist [mm] e^{z^n} \ne (e^z)^n [/mm] (= [mm] e^{zn} [/mm]

Zum Beispiel ist [mm] e^{(-i*\pi)^2}=e^{- \pi^2}. [/mm]


>  
> >  

> > Dein Ansatz ist aber ok… allerdings ist mir aufgefallen,
> > dass meine Aussage zu Beginn falsch war, dass der
> > Integrationsbereich keine Rolle spielt… zumindest für
> > die Lösung.
> > Du müsstest jetzt bei [mm]e^{-in\pi}[/mm] nämlich zwischen geraden
> > und ungeraden n unterscheiden.
>  
> Also hier müsste ich nochmal weitermachen... richtig?

Ja


>  
> >  Diese Unterscheidung fällt praktischerweise beim

> > Integrationsbereich von 0 bis [mm]2\pi[/mm] weg, weil der Faktor 2
> > da immer drin vorkommt.
> >
> > D.h. bezugnehmend auf meinen Kommentar solltest du
> > korrekterweise [mm]c_n = \bruch{1}{2\pi}\cdot{}\integral_{0}^{2\pi}{x\cdot{}e^{-inx dx}[/mm]
> >   berechnen… das war ja auch die Aufgabenstellung.

>  >  
> > Gruß,
>  >  Gono  
>
> Naja, die Aufgabenstellung war schon das Intervall
> [mm][-\pi;\pi][/mm] .
>  


Bezug
                                                
Bezug
Komplexe Fourier-Reihe: Frage (beantwortet)
Status: (Frage) beantwortet Status 
Datum: 10:32 Do 01.11.2018
Autor: hase-hh

Moin!
> >  

> > Naja, ich weiß nicht, wie ich die Grenzen da angeben kann
> > (Formatierungsproblem)   ... [mm][-\pi;\pi][/mm]  ist jeweils
> > gemeint...
>
> [mm]F(b)-F(a)=[F(x)]_a^b[/mm]
>  
> meinst Du das ?
>  >

Ja, genau  [mm] ]_a^b [/mm]
    

> > > > [mm]c_n[/mm] = [mm]\bruch{1}{2\pi}*( [-\bruch{1}{in}*x*e^{-inx}][/mm] -

> > > > [mm]c_n[/mm] = [mm]\bruch{1}{2\pi}*(-\bruch{1}{in}*\pi*e^{(-i*\pi)^n} -(-\bruch{1}{in}*(-\pi)*e^{-i*(-\pi)^n}))[/mm]
> > > > - [mm][\bruch{1}{i^2n^2}*e^{-inx}][/mm]
>  >  >  Und hier wird aus dem [mm]i\pi n[/mm] im Exponent plötzlich
> ein
> > > [mm]i\pi^n[/mm]… wieso?
>  >  
> > Ich habe [mm]e^{-i*\pi*n}[/mm] umgeformt zu [mm]e^{(-i*\pi)^n}[/mm] ...weil
> > [mm]e^{-i*\pi}[/mm] = -1  bzw. [mm]e^{i*\pi}[/mm] = -1  ist, oder nicht?
>  
> Upps ! Es ist [mm]e^{z^n} \ne (e^z)^n[/mm] (= [mm]e^{zn}[/mm]
>  
> Zum Beispiel ist [mm]e^{(-i*\pi)^2}=e^{- \pi^2}.[/mm]
>  

Ok, wie geht das aber dann mit dem Einheitskreis?

Bisher habe ich gelernt  [mm] e^{-i*\pi} [/mm] = -1.

Was ist aber dann [mm] e^{-i*\pi*n} [/mm] ?

In meinen Unterlagen finde ich  [mm] (-1)^n, [/mm] aber wie komme ich dahin???














Bezug
                                                        
Bezug
Komplexe Fourier-Reihe: Potenzgesetz
Status: (Antwort) fertig Status 
Datum: 10:48 Do 01.11.2018
Autor: Loddar

Hallo Hase,

gemäß MBPotenzgesetz [mm] $a^{m*n} [/mm] \ = \ [mm] \left( \ a^m \right)^n$ [/mm] gilt:

[mm] $e^{-i*\pi*n} [/mm] \ = \ [mm] \left( \ e^{-i*\pi} \ \right)^n [/mm] \ = \ [mm] (-1)^n$ [/mm]


Nun klar(er)?


Gruß
Loddar

Bezug
                                                                
Bezug
Komplexe Fourier-Reihe: Mitteilung
Status: (Mitteilung) Reaktion unnötig Status 
Datum: 14:34 Fr 02.11.2018
Autor: hase-hh


> Hallo Hase,
>  
> gemäß MBPotenzgesetz [mm]a^{m*n} \ = \ \left( \ a^m \right)^n[/mm]
> gilt:
>  
> [mm]e^{-i*\pi*n} \ = \ \left( \ e^{-i*\pi} \ \right)^n \ = \ (-1)^n[/mm]
>  
>
> Nun klar(er)?
>  
>
> Gruß
>  Loddar


Ja, vielen Dank! Kurz und knackig!

Also war mein Fehler, dass ich weiter oben die Klammern um die Basis nicht gesetzt habe...




Bezug
                                        
Bezug
Komplexe Fourier-Reihe: Antwort
Status: (Antwort) fertig Status 
Datum: 17:12 Mi 31.10.2018
Autor: Gonozal_IX

Hiho,

nur der Vollständigkeit halber:

> Naja, die Aufgabenstellung war schon das Intervall [mm][-\pi;\pi][/mm] .

also in dem Video, was du gepostet hast, nicht. Da ging es um das Intervall [mm] $[0,2\pi]$ [/mm]

Gruß,
Gono

Bezug
                                                
Bezug
Komplexe Fourier-Reihe: Mitteilung
Status: (Mitteilung) Reaktion unnötig Status 
Datum: 09:59 Do 01.11.2018
Autor: hase-hh


> Hiho,
>  
> nur der Vollständigkeit halber:
>  
> > Naja, die Aufgabenstellung war schon das Intervall
> [mm][-\pi;\pi][/mm] .
>  also in dem Video, was du gepostet hast, nicht. Da ging es
> um das Intervall [mm][0,2\pi][/mm]
>  
> Gruß,
>  Gono

Ja, im Video schon, aber was Besseres habe ich nicht gefunden.  

Die Aufgabenstellung betrachtet das Intervall [- [mm] \pi;\pi] [/mm] .

Amen.

Bezug
                                                
Bezug
Komplexe Fourier-Reihe: Frage (überfällig)
Status: (Frage) überfällig Status 
Datum: 14:00 So 04.11.2018
Autor: hase-hh

Moin Moin,

ich fasse nochmal zusammen...


2. Ich berechne die [mm]c_n[/mm]


[mm]c_n[/mm] = [mm]\bruch{1}{2\pi}*\integral_{-\pi}^{\pi}{x*e^{-inx} dx}[/mm]


Ich wähle für die partielle Integration

u = x       v' = [mm]e^{-inx}[/mm]
  
u' = 1      v = [mm]-\bruch{1}{in}*e^{-inx}[/mm]
  
[mm] c_n [/mm] = [mm] \bruch{1}{2\pi}*( [-\bruch{1}{in}*x*e^{-inx}]_{-\pi}^\pi [/mm] - [mm]\integral_{-\pi}^{\pi}{-\bruch{1}{in}*e^{-inx} dx})[/mm]

[mm]c_n[/mm] = [mm]\bruch{1}{2\pi}*( [-\bruch{1}{in}*x*e^{-inx}]_{-\pi}^\pi}[/mm] - [mm][(-\bruch{1}{in})^2*e^{-inx}]_{-\pi}^\pi)[/mm]

[mm]c_n[/mm] = [mm]\bruch{1}{2\pi}*( (-\bruch{1}{in}*\pi*(e^{-i*\pi})^n}) -(-\bruch{1}{in}*(-\pi)*(e^{-i*(-\pi)})^n})[/mm] - [mm] [\bruch{1}{i^2n^2}*e^{-inx}]_{-\pi}^\pi [/mm] )

[mm]c_n[/mm] = [mm]\bruch{1}{2\pi}*(-\bruch{\pi}{in}*(-1)^n (-\bruch{-\pi}{in}*(-1)^n))[/mm] - [mm] \bruch{1}{2\pi}*[\bruch{1}{-1*n^2}*e^{-inx}]_{-\pi}^\pi [/mm]
  
[mm] c_n [/mm] = [mm] \bruch{1}{2\pi}*(-\bruch{\pi}{in}*(-1)^n -(\bruch{\pi}{in}*(-1)^n)) +\bruch{1}{2\pi}*(\bruch{1}{n^2}*(e^{-i*\pi})^n [/mm] - [mm] \bruch{1}{n^2}*(e^{-i*(-\pi)})^n [/mm] )

[mm] c_n [/mm] = [mm] \bruch{1}{2\pi}*(-\bruch{2\pi}{in}*(-1)^n [/mm] ) [mm] +\bruch{1}{2\pi}*(\bruch{1}{n^2}*(-1)^n [/mm] - [mm] \bruch{1}{n^2}*(-1)^n [/mm] )

[mm] c_n [/mm] = [mm] \bruch{1}{2\pi}*(-\bruch{2\pi}{in}*(-1)^n [/mm] + 0

[mm] c_n [/mm] = - [mm] \bruch{1}{in}*(-1)^n [/mm]


richtig?


Wie kann ich jetzt die Unterscheidung zwischen ungeraden n und geraden n machen?

keine Idee!



Bezug
                                                        
Bezug
Komplexe Fourier-Reihe: Fälligkeit abgelaufen
Status: (Mitteilung) Reaktion unnötig Status 
Datum: 14:20 Di 06.11.2018
Autor: matux

$MATUXTEXT(ueberfaellige_frage)
Bezug
                                
Bezug
Komplexe Fourier-Reihe: Frage (überfällig)
Status: (Frage) überfällig Status 
Datum: 15:12 Fr 02.11.2018
Autor: hase-hh

Moin Moin,

ich möchte die Lösung noch etwas vervollständigen...

> > 2. Ich berechne die [mm]c_n[/mm]
> >
> > [mm]c_n[/mm] = [mm]\bruch{1}{2\pi}*\integral_{-\pi}^{\pi}{x*e^{-inx} dx}[/mm]

> >
> > Ich wähle für die partielle Integration
> >
> > u = x       v' = [mm]e^{-inx}[/mm]
>  >  
> > u' = 1      v = [mm]-\bruch{1}{in}*e^{-inx}[/mm]
>  [ok]
>    

[mm]c_n[/mm] = [mm]\bruch{1}{2\pi}*( [-\bruch{1}{in}*x*e^{-inx}]_{-\pi}^{\pi} [/mm] - [mm]\integral_{-\pi}^{\pi}{-\bruch{1}{in}*e^{-inx} dx})[/mm]

[mm]c_n[/mm] = [mm]\bruch{1}{2\pi}*( [-\bruch{1}{in}*x*e^{-inx}]_{-\pi}^{\pi}[/mm] - [mm][(-\bruch{1}{in})^2*e^{-inx}]_{-\pi}^{\pi})[/mm]

[mm]c_n[/mm] = [mm]\bruch{1}{2\pi}*(-\bruch{1}{in}*\pi*(e^{-i*\pi})^n -(-\bruch{1}{in}*(-\pi)*(e^{-i*(-\pi)})^n)[/mm] - [mm](\bruch{1}{i^2n^2}*(e^{-i*\pi})^n - \bruch{1}{i^2n^2}*(e^{-i*(-\pi)})^n)[/mm]

[mm]c_n[/mm] = [mm]\bruch{1}{2\pi}*(-\bruch{1}{in}*\pi*(-1})^n -(-\bruch{1}{in}*(-\pi)*(-1)^n)[/mm] - [mm](\bruch{1}{i^2n^2}*(-1)^n - \bruch{1}{i^2n^2}*(-1)^n)[/mm]


[mm]c_n[/mm] = [mm]\bruch{1}{2\pi}*(-\bruch{1}{in}*\pi*(-1})^n -(\bruch{1}{in}*(\pi)*(-1)^n)[/mm] - 0

[mm]c_n[/mm] = [mm]\bruch{1}{2\pi}*(-\bruch{2}{in}*\pi*(-1})^n) [/mm]

[mm]c_n[/mm] = [mm]-\bruch{1}{in}*(-1})^n [/mm]

Ist das soweit korrekt?


Muss ich jetzt zwischen geraden und ungeraden n unterscheiden, und wie würde dann meine Lösung aussehen?

Hmm...  ginge das so:

n = 2k    

n = 2k+1


[mm] c_{2k}= -\bruch{1}{i*2k}*(-1)^{2k} [/mm] =  [mm] -\bruch{1}{2ik} [/mm]

[mm] c_{2k+1}= -\bruch{1}{i*(2k+1)}*(-1)^{2k+1} [/mm] = [mm] \bruch{1}{i*(2k+1)} [/mm]

???

Bezug
                                        
Bezug
Komplexe Fourier-Reihe: Fälligkeit abgelaufen
Status: (Mitteilung) Reaktion unnötig Status 
Datum: 15:20 So 04.11.2018
Autor: matux

$MATUXTEXT(ueberfaellige_frage)
Bezug
                                
Bezug
Komplexe Fourier-Reihe: Frage (beantwortet)
Status: (Frage) beantwortet Status 
Datum: 08:04 Mi 07.11.2018
Autor: hase-hh

Moin Moin,

ich möchte die Lösung noch etwas vervollständigen...

> > 2. Ich berechne die [mm]c_n[/mm]
> >
> > [mm]c_n[/mm] = [mm]\bruch{1}{2\pi}*\integral_{-\pi}^{\pi}{x*e^{-inx} dx}[/mm]

> >
> > Ich wähle für die partielle Integration
> >
> > u = x       v' = [mm]e^{-inx}[/mm]
>  >  
> > u' = 1      v = [mm]-\bruch{1}{in}*e^{-inx}[/mm]
>  [ok]
>    

[mm]c_n[/mm] = [mm]\bruch{1}{2\pi}*( [-\bruch{1}{in}*x*e^{-inx}]_{-\pi}^{\pi} [/mm] - [mm]\integral_{-\pi}^{\pi}{-\bruch{1}{in}*e^{-inx} dx})[/mm]

[mm]c_n[/mm] = [mm]\bruch{1}{2\pi}*( [-\bruch{1}{in}*x*e^{-inx}]_{-\pi}^{\pi}[/mm] - [mm][(-\bruch{1}{in})^2*e^{-inx}]_{-\pi}^{\pi})[/mm]

[mm]c_n[/mm] = [mm]\bruch{1}{2\pi}*(-\bruch{1}{in}*\pi*(e^{-i*\pi})^n -(-\bruch{1}{in}*(-\pi)*(e^{-i*(-\pi)})^n)[/mm] - [mm](\bruch{1}{i^2n^2}*(e^{-i*\pi})^n - \bruch{1}{i^2n^2}*(e^{-i*(-\pi)})^n)[/mm]

[mm]c_n[/mm] = [mm]\bruch{1}{2\pi}*(-\bruch{1}{in}*\pi*(-1})^n -(-\bruch{1}{in}*(-\pi)*(-1)^n)[/mm] - [mm](\bruch{1}{i^2n^2}*(-1)^n - \bruch{1}{i^2n^2}*(-1)^n)[/mm]


[mm]c_n[/mm] = [mm]\bruch{1}{2\pi}*(-\bruch{1}{in}*\pi*(-1})^n -(\bruch{1}{in}*(\pi)*(-1)^n)[/mm] - 0

[mm]c_n[/mm] = [mm]\bruch{1}{2\pi}*(-\bruch{2}{in}*\pi*(-1})^n) [/mm]

[mm]c_n[/mm] = [mm]-\bruch{1}{in}*(-1})^n [/mm]

Ist das soweit korrekt?


Muss ich jetzt zwischen geraden und ungeraden n unterscheiden, und wie würde dann meine Lösung aussehen?

Hmm...  ginge das so:

n = 2k    

n = 2k+1


[mm] c_{2k}= -\bruch{1}{i*2k}*(-1)^{2k} [/mm] =  [mm] -\bruch{1}{2ik} [/mm]

[mm] c_{2k+1}= -\bruch{1}{i*(2k+1)}*(-1)^{2k+1} [/mm] = [mm] \bruch{1}{i*(2k+1)} [/mm]

???

Bezug
                                        
Bezug
Komplexe Fourier-Reihe: Mitteilung
Status: (Mitteilung) Reaktion unnötig Status 
Datum: 08:45 Fr 09.11.2018
Autor: hase-hh

Die Frage ist nach wie vor offen. :(

Bezug
                                        
Bezug
Komplexe Fourier-Reihe: Antwort
Status: (Antwort) fertig Status 
Datum: 09:57 Fr 09.11.2018
Autor: fred97




$ [mm] c_{2k}= -\bruch{1}{i\cdot{}2k}\cdot{}(-1)^{2k} [/mm] $ =  $ [mm] -\bruch{1}{2ik} [/mm] $

$ [mm] c_{2k+1}= -\bruch{1}{i\cdot{}(2k+1)}\cdot{}(-1)^{2k+1} [/mm] $ = $ [mm] \bruch{1}{i\cdot{}(2k+1)} [/mm] $

ist richtig.


Bezug
                                                
Bezug
Komplexe Fourier-Reihe: Mitteilung
Status: (Mitteilung) Reaktion unnötig Status 
Datum: 19:11 Fr 09.11.2018
Autor: hase-hh

Dankeschön!!

Bezug
Ansicht: [ geschachtelt ] | ^ Forum "Fourier-Transformation"  | ^^ Alle Foren  | ^ Forenbaum  | Materialien


^ Seitenanfang ^
www.unimatheforum.de
[ Startseite | Forum | Wissen | Kurse | Mitglieder | Team | Impressum ]